Finding two disjoint subsets whose sum is below 1/2












1












$begingroup$


We are given $2k+1$ positive numbers whose sum is $1$. Can we always find two disjoint subsets of $k$ numbers such that the sum of each subset is at most $1/2$?



When $k=1$, this is easy: we have $3$ positive numbers whose sum is $1$, so at most one number is more than $1/2$, so at least two numbers are at most $1/2$.
Is it true for $k>1$?










share|cite|improve this question









$endgroup$

















    1












    $begingroup$


    We are given $2k+1$ positive numbers whose sum is $1$. Can we always find two disjoint subsets of $k$ numbers such that the sum of each subset is at most $1/2$?



    When $k=1$, this is easy: we have $3$ positive numbers whose sum is $1$, so at most one number is more than $1/2$, so at least two numbers are at most $1/2$.
    Is it true for $k>1$?










    share|cite|improve this question









    $endgroup$















      1












      1








      1





      $begingroup$


      We are given $2k+1$ positive numbers whose sum is $1$. Can we always find two disjoint subsets of $k$ numbers such that the sum of each subset is at most $1/2$?



      When $k=1$, this is easy: we have $3$ positive numbers whose sum is $1$, so at most one number is more than $1/2$, so at least two numbers are at most $1/2$.
      Is it true for $k>1$?










      share|cite|improve this question









      $endgroup$




      We are given $2k+1$ positive numbers whose sum is $1$. Can we always find two disjoint subsets of $k$ numbers such that the sum of each subset is at most $1/2$?



      When $k=1$, this is easy: we have $3$ positive numbers whose sum is $1$, so at most one number is more than $1/2$, so at least two numbers are at most $1/2$.
      Is it true for $k>1$?







      combinatorics






      share|cite|improve this question













      share|cite|improve this question











      share|cite|improve this question




      share|cite|improve this question










      asked Dec 18 '18 at 21:30









      Erel Segal-HaleviErel Segal-Halevi

      4,33011861




      4,33011861






















          3 Answers
          3






          active

          oldest

          votes


















          1












          $begingroup$

          Note: assuming the sets exist, it is clear that their union is all of the points with one exception. We might as well assume that the missing point is the largest (if not, then the largest is in one of the two sets and we can swap it out for the missing element without hurting the desired sum).



          Work inductively. Suppose we are done for the case $k-1$. Consider the case corresponding to $k$. Order our $2k+1$ numbers as $a_1≤a_2≤cdots ≤a_{2k+1}$ . Now consider the $2k-1$ points ${a_1,cdots, a_{2k-1}}$. Of course these sum to $1-a_{2k+1}-a_{2k}$. Define ${b_i}_{i=1}^{2k-1}$ by $b_i=frac {a_i}{1-a_{2k+1}-a_{2k}}$ Now the $b_i$ form a set of $2k-1$ numbers that sum to $1$ so we can apply the inductive hypothesis to this set. We get a partition of $b_1,cdots, b_{2k-2}$ into two sets each of which sums to less than $frac 12$. Multiplying by $1-a_{2k+1}-a_{2k}$ we get a partition of $a_1,cdots, a_{2k-2}$ into two sets each of which sums to less than $frac {1-a_{2k+1}-a_{2k}}2$. Call these two sets $S_1,S_2$. We now claim that we can adjoin $a_{2k-1}$ to one of them and $a_{2k}$ to the other to get the desired partition of size $k$.



          To see this, suppose we add $a_{2k}$ to $S_1$. Then the new sum is less than $$frac {1-a_{2k}-a_{2k+1}}2+a_{2k}=frac {1+a_{2k}-a_{2k+1}}2$$ But by the ordering, $a_{2k}≤a_{2k+1}$ so the numerator here is at most $1$ and we are done. Of course $a_{2k-1}$ is not greater than $a_{2k}$ so the same argument works for $S_2$.






          share|cite|improve this answer











          $endgroup$













          • $begingroup$
            Great answer, thanks! By what name can I cite you in a paper?
            $endgroup$
            – Erel Segal-Halevi
            Dec 19 '18 at 3:57










          • $begingroup$
            No need for a citation, thanks. Appreciate the thought!
            $endgroup$
            – lulu
            Dec 19 '18 at 11:17



















          2












          $begingroup$

          Based on the two previous answers, here is a simpler algorithm.



          Order the $2k+1$ numbers in increasing order, $a_1leqcdots leq a_{2k+1}$.



          Consider first the $k$ numbers with even indices: each number in this set is weakly smaller than the number immediately after it, which is not in this set. Hence, the sum of these numbers must be at most $1/2$.



          Consider next the $k$ numbers with odd indices, except the last one: again each number in this set is weakly smaller than the number immediately after it, which is not in this set. Hence, the sum of these numbers must be at most $1/2$ too.






          share|cite|improve this answer









          $endgroup$





















            1












            $begingroup$

            Yes, here's an explicit algorithm for constructing two disjoint $k$-subsets.



            Let $H$ be a family/multiset of positive numbers indexed by $mathbb{N}_{le(2k+1)}$, with $mathbb{N}$ excluding zero.



            Let's further constrain $H$ to be weakly monotonically increasing in the index, so $H_{2k+1}$ is the co-largest element.



            Suppose $H_{2k+1} ge frac{1}{2}$ , then $H_{(1cdots k)}$ and $H_{(k+1 cdots 2k)}$ both have sum $le frac{1}{2}$ and we're done.



            Suppose $H_{2k+1} le frac{1}{2}$ , then let's define the champion $k$-subfamily $c$ and its set of indices $gamma$ by the following algorithm.




            1. The very first thing we do is exclude the largest index $2k+1$ (which corresponds to the greatest element of $H$), none of the candidate subsets involve this index.

            2. Enumerate all the $k$-subsets of $mathbb{N}_{le(2k)}$ , call the subset under consideration $w$

            3. reject all $w$'s for which $sum_{j in w}H_j$ is greater than or equal to $frac{1}{2}$ .

            4. reject all $w$'s that do not have the maximum possible sum of any of the remaining $w$'s.

            5. Of the remaining sets, pick the greatest remaining set by comparing the greatest indices one at a time of $w$ . For instance ${5,4,1}$ beats ${5,3,1}$ because $4 > 3$ . This tiebreaker is completely arbitrary, it's just there to guarantee uniqueness.


            6. $c$ is $w$ and $gamma$ is $H_c$ .


            Note that at least one subset $w = {1, cdots, k}$ makes it past step 3, so the algorithm always produces a unique champion subset of indices $c$, which corresponds to a multiset $gamma = H_c$.



            Let's define multiset indexed by the champion complement set $H[(mathbb{N}le 2*k) setminus c)]$. Henceforth $d = (mathbb{N}le 2*k) setminus c$, and $delta = H_d$ .



            $gamma$ has sum less than $frac{1}{2}$ by construction. We will show that $delta$ also has sum less than $frac{1}{2}$ .





            If $sum delta = sum_{j in d}H_j lt frac{1}{2}$, then we have found a pair of disjoint subsets $c$ and $d$ each with sum less than $frac{1}{2}$, as required.





            If $sum delta = frac{1}{2}$, then then the sum of $sum gamma$ and $H_{2k+1}$ is $frac{1}{2}$ .



            We will define an operation in terms of $gamma$ and $delta$ to produce a new multiset $gamma'$ with a larger sum than $gamma$, but still less than $frac{1}{2}$ .



            Pick the largest element of $delta$, tie-breaking on the index if necessary. Call its index $t in d$ . Pick the largest element of $gamma$ less than $t$ , call its index $s in c$ .



            If there is no $s in c$, then $sum delta le sum gamma$. $sum gamma < frac{1}{2}$ by construction, so there's a contradiction. Therefore there is an $s in c$ less than $t in d$ .



            $t-s$ is positive by construction. $t-s$ is also less than $H_{2k+1}$ because it is a difference of two positive numbers, each of which is smaller than $H_{2k+1}$ .



            Define $gamma'$ as the multiset obtained by removing the element with index $s$ from $gamma$ and adding the element with index $t$ from $delta$ . $gamma'$ is a better candidate for champion than $gamma$ . Therefore $gamma$ is not champion.





            If $sum delta gt frac{1}{2}$, then we can repeat the construction defined above for producing $gamma'$ . $gamma'$ has sum less than $frac{1}{2}$ and is a better candidate for champion set than $gamma$ . Therefore $gamma$ is not champion.






            share|cite|improve this answer











            $endgroup$













            • $begingroup$
              What is "co-largest"?
              $endgroup$
              – Erel Segal-Halevi
              Dec 19 '18 at 3:59










            • $begingroup$
              "one of the largest". $gamma$ and $delta$ are multisets so there might be multiple largest items. It's overloading the co- prefix, but I have heard it before. Is there a better way to say it?
              $endgroup$
              – Gregory Nisbet
              Dec 19 '18 at 4:13













            Your Answer





            StackExchange.ifUsing("editor", function () {
            return StackExchange.using("mathjaxEditing", function () {
            StackExchange.MarkdownEditor.creationCallbacks.add(function (editor, postfix) {
            StackExchange.mathjaxEditing.prepareWmdForMathJax(editor, postfix, [["$", "$"], ["\\(","\\)"]]);
            });
            });
            }, "mathjax-editing");

            StackExchange.ready(function() {
            var channelOptions = {
            tags: "".split(" "),
            id: "69"
            };
            initTagRenderer("".split(" "), "".split(" "), channelOptions);

            StackExchange.using("externalEditor", function() {
            // Have to fire editor after snippets, if snippets enabled
            if (StackExchange.settings.snippets.snippetsEnabled) {
            StackExchange.using("snippets", function() {
            createEditor();
            });
            }
            else {
            createEditor();
            }
            });

            function createEditor() {
            StackExchange.prepareEditor({
            heartbeatType: 'answer',
            autoActivateHeartbeat: false,
            convertImagesToLinks: true,
            noModals: true,
            showLowRepImageUploadWarning: true,
            reputationToPostImages: 10,
            bindNavPrevention: true,
            postfix: "",
            imageUploader: {
            brandingHtml: "Powered by u003ca class="icon-imgur-white" href="https://imgur.com/"u003eu003c/au003e",
            contentPolicyHtml: "User contributions licensed under u003ca href="https://creativecommons.org/licenses/by-sa/3.0/"u003ecc by-sa 3.0 with attribution requiredu003c/au003e u003ca href="https://stackoverflow.com/legal/content-policy"u003e(content policy)u003c/au003e",
            allowUrls: true
            },
            noCode: true, onDemand: true,
            discardSelector: ".discard-answer"
            ,immediatelyShowMarkdownHelp:true
            });


            }
            });














            draft saved

            draft discarded


















            StackExchange.ready(
            function () {
            StackExchange.openid.initPostLogin('.new-post-login', 'https%3a%2f%2fmath.stackexchange.com%2fquestions%2f3045731%2ffinding-two-disjoint-subsets-whose-sum-is-below-1-2%23new-answer', 'question_page');
            }
            );

            Post as a guest















            Required, but never shown

























            3 Answers
            3






            active

            oldest

            votes








            3 Answers
            3






            active

            oldest

            votes









            active

            oldest

            votes






            active

            oldest

            votes









            1












            $begingroup$

            Note: assuming the sets exist, it is clear that their union is all of the points with one exception. We might as well assume that the missing point is the largest (if not, then the largest is in one of the two sets and we can swap it out for the missing element without hurting the desired sum).



            Work inductively. Suppose we are done for the case $k-1$. Consider the case corresponding to $k$. Order our $2k+1$ numbers as $a_1≤a_2≤cdots ≤a_{2k+1}$ . Now consider the $2k-1$ points ${a_1,cdots, a_{2k-1}}$. Of course these sum to $1-a_{2k+1}-a_{2k}$. Define ${b_i}_{i=1}^{2k-1}$ by $b_i=frac {a_i}{1-a_{2k+1}-a_{2k}}$ Now the $b_i$ form a set of $2k-1$ numbers that sum to $1$ so we can apply the inductive hypothesis to this set. We get a partition of $b_1,cdots, b_{2k-2}$ into two sets each of which sums to less than $frac 12$. Multiplying by $1-a_{2k+1}-a_{2k}$ we get a partition of $a_1,cdots, a_{2k-2}$ into two sets each of which sums to less than $frac {1-a_{2k+1}-a_{2k}}2$. Call these two sets $S_1,S_2$. We now claim that we can adjoin $a_{2k-1}$ to one of them and $a_{2k}$ to the other to get the desired partition of size $k$.



            To see this, suppose we add $a_{2k}$ to $S_1$. Then the new sum is less than $$frac {1-a_{2k}-a_{2k+1}}2+a_{2k}=frac {1+a_{2k}-a_{2k+1}}2$$ But by the ordering, $a_{2k}≤a_{2k+1}$ so the numerator here is at most $1$ and we are done. Of course $a_{2k-1}$ is not greater than $a_{2k}$ so the same argument works for $S_2$.






            share|cite|improve this answer











            $endgroup$













            • $begingroup$
              Great answer, thanks! By what name can I cite you in a paper?
              $endgroup$
              – Erel Segal-Halevi
              Dec 19 '18 at 3:57










            • $begingroup$
              No need for a citation, thanks. Appreciate the thought!
              $endgroup$
              – lulu
              Dec 19 '18 at 11:17
















            1












            $begingroup$

            Note: assuming the sets exist, it is clear that their union is all of the points with one exception. We might as well assume that the missing point is the largest (if not, then the largest is in one of the two sets and we can swap it out for the missing element without hurting the desired sum).



            Work inductively. Suppose we are done for the case $k-1$. Consider the case corresponding to $k$. Order our $2k+1$ numbers as $a_1≤a_2≤cdots ≤a_{2k+1}$ . Now consider the $2k-1$ points ${a_1,cdots, a_{2k-1}}$. Of course these sum to $1-a_{2k+1}-a_{2k}$. Define ${b_i}_{i=1}^{2k-1}$ by $b_i=frac {a_i}{1-a_{2k+1}-a_{2k}}$ Now the $b_i$ form a set of $2k-1$ numbers that sum to $1$ so we can apply the inductive hypothesis to this set. We get a partition of $b_1,cdots, b_{2k-2}$ into two sets each of which sums to less than $frac 12$. Multiplying by $1-a_{2k+1}-a_{2k}$ we get a partition of $a_1,cdots, a_{2k-2}$ into two sets each of which sums to less than $frac {1-a_{2k+1}-a_{2k}}2$. Call these two sets $S_1,S_2$. We now claim that we can adjoin $a_{2k-1}$ to one of them and $a_{2k}$ to the other to get the desired partition of size $k$.



            To see this, suppose we add $a_{2k}$ to $S_1$. Then the new sum is less than $$frac {1-a_{2k}-a_{2k+1}}2+a_{2k}=frac {1+a_{2k}-a_{2k+1}}2$$ But by the ordering, $a_{2k}≤a_{2k+1}$ so the numerator here is at most $1$ and we are done. Of course $a_{2k-1}$ is not greater than $a_{2k}$ so the same argument works for $S_2$.






            share|cite|improve this answer











            $endgroup$













            • $begingroup$
              Great answer, thanks! By what name can I cite you in a paper?
              $endgroup$
              – Erel Segal-Halevi
              Dec 19 '18 at 3:57










            • $begingroup$
              No need for a citation, thanks. Appreciate the thought!
              $endgroup$
              – lulu
              Dec 19 '18 at 11:17














            1












            1








            1





            $begingroup$

            Note: assuming the sets exist, it is clear that their union is all of the points with one exception. We might as well assume that the missing point is the largest (if not, then the largest is in one of the two sets and we can swap it out for the missing element without hurting the desired sum).



            Work inductively. Suppose we are done for the case $k-1$. Consider the case corresponding to $k$. Order our $2k+1$ numbers as $a_1≤a_2≤cdots ≤a_{2k+1}$ . Now consider the $2k-1$ points ${a_1,cdots, a_{2k-1}}$. Of course these sum to $1-a_{2k+1}-a_{2k}$. Define ${b_i}_{i=1}^{2k-1}$ by $b_i=frac {a_i}{1-a_{2k+1}-a_{2k}}$ Now the $b_i$ form a set of $2k-1$ numbers that sum to $1$ so we can apply the inductive hypothesis to this set. We get a partition of $b_1,cdots, b_{2k-2}$ into two sets each of which sums to less than $frac 12$. Multiplying by $1-a_{2k+1}-a_{2k}$ we get a partition of $a_1,cdots, a_{2k-2}$ into two sets each of which sums to less than $frac {1-a_{2k+1}-a_{2k}}2$. Call these two sets $S_1,S_2$. We now claim that we can adjoin $a_{2k-1}$ to one of them and $a_{2k}$ to the other to get the desired partition of size $k$.



            To see this, suppose we add $a_{2k}$ to $S_1$. Then the new sum is less than $$frac {1-a_{2k}-a_{2k+1}}2+a_{2k}=frac {1+a_{2k}-a_{2k+1}}2$$ But by the ordering, $a_{2k}≤a_{2k+1}$ so the numerator here is at most $1$ and we are done. Of course $a_{2k-1}$ is not greater than $a_{2k}$ so the same argument works for $S_2$.






            share|cite|improve this answer











            $endgroup$



            Note: assuming the sets exist, it is clear that their union is all of the points with one exception. We might as well assume that the missing point is the largest (if not, then the largest is in one of the two sets and we can swap it out for the missing element without hurting the desired sum).



            Work inductively. Suppose we are done for the case $k-1$. Consider the case corresponding to $k$. Order our $2k+1$ numbers as $a_1≤a_2≤cdots ≤a_{2k+1}$ . Now consider the $2k-1$ points ${a_1,cdots, a_{2k-1}}$. Of course these sum to $1-a_{2k+1}-a_{2k}$. Define ${b_i}_{i=1}^{2k-1}$ by $b_i=frac {a_i}{1-a_{2k+1}-a_{2k}}$ Now the $b_i$ form a set of $2k-1$ numbers that sum to $1$ so we can apply the inductive hypothesis to this set. We get a partition of $b_1,cdots, b_{2k-2}$ into two sets each of which sums to less than $frac 12$. Multiplying by $1-a_{2k+1}-a_{2k}$ we get a partition of $a_1,cdots, a_{2k-2}$ into two sets each of which sums to less than $frac {1-a_{2k+1}-a_{2k}}2$. Call these two sets $S_1,S_2$. We now claim that we can adjoin $a_{2k-1}$ to one of them and $a_{2k}$ to the other to get the desired partition of size $k$.



            To see this, suppose we add $a_{2k}$ to $S_1$. Then the new sum is less than $$frac {1-a_{2k}-a_{2k+1}}2+a_{2k}=frac {1+a_{2k}-a_{2k+1}}2$$ But by the ordering, $a_{2k}≤a_{2k+1}$ so the numerator here is at most $1$ and we are done. Of course $a_{2k-1}$ is not greater than $a_{2k}$ so the same argument works for $S_2$.







            share|cite|improve this answer














            share|cite|improve this answer



            share|cite|improve this answer








            edited Dec 19 '18 at 11:18

























            answered Dec 18 '18 at 22:04









            lulululu

            42.5k25080




            42.5k25080












            • $begingroup$
              Great answer, thanks! By what name can I cite you in a paper?
              $endgroup$
              – Erel Segal-Halevi
              Dec 19 '18 at 3:57










            • $begingroup$
              No need for a citation, thanks. Appreciate the thought!
              $endgroup$
              – lulu
              Dec 19 '18 at 11:17


















            • $begingroup$
              Great answer, thanks! By what name can I cite you in a paper?
              $endgroup$
              – Erel Segal-Halevi
              Dec 19 '18 at 3:57










            • $begingroup$
              No need for a citation, thanks. Appreciate the thought!
              $endgroup$
              – lulu
              Dec 19 '18 at 11:17
















            $begingroup$
            Great answer, thanks! By what name can I cite you in a paper?
            $endgroup$
            – Erel Segal-Halevi
            Dec 19 '18 at 3:57




            $begingroup$
            Great answer, thanks! By what name can I cite you in a paper?
            $endgroup$
            – Erel Segal-Halevi
            Dec 19 '18 at 3:57












            $begingroup$
            No need for a citation, thanks. Appreciate the thought!
            $endgroup$
            – lulu
            Dec 19 '18 at 11:17




            $begingroup$
            No need for a citation, thanks. Appreciate the thought!
            $endgroup$
            – lulu
            Dec 19 '18 at 11:17











            2












            $begingroup$

            Based on the two previous answers, here is a simpler algorithm.



            Order the $2k+1$ numbers in increasing order, $a_1leqcdots leq a_{2k+1}$.



            Consider first the $k$ numbers with even indices: each number in this set is weakly smaller than the number immediately after it, which is not in this set. Hence, the sum of these numbers must be at most $1/2$.



            Consider next the $k$ numbers with odd indices, except the last one: again each number in this set is weakly smaller than the number immediately after it, which is not in this set. Hence, the sum of these numbers must be at most $1/2$ too.






            share|cite|improve this answer









            $endgroup$


















              2












              $begingroup$

              Based on the two previous answers, here is a simpler algorithm.



              Order the $2k+1$ numbers in increasing order, $a_1leqcdots leq a_{2k+1}$.



              Consider first the $k$ numbers with even indices: each number in this set is weakly smaller than the number immediately after it, which is not in this set. Hence, the sum of these numbers must be at most $1/2$.



              Consider next the $k$ numbers with odd indices, except the last one: again each number in this set is weakly smaller than the number immediately after it, which is not in this set. Hence, the sum of these numbers must be at most $1/2$ too.






              share|cite|improve this answer









              $endgroup$
















                2












                2








                2





                $begingroup$

                Based on the two previous answers, here is a simpler algorithm.



                Order the $2k+1$ numbers in increasing order, $a_1leqcdots leq a_{2k+1}$.



                Consider first the $k$ numbers with even indices: each number in this set is weakly smaller than the number immediately after it, which is not in this set. Hence, the sum of these numbers must be at most $1/2$.



                Consider next the $k$ numbers with odd indices, except the last one: again each number in this set is weakly smaller than the number immediately after it, which is not in this set. Hence, the sum of these numbers must be at most $1/2$ too.






                share|cite|improve this answer









                $endgroup$



                Based on the two previous answers, here is a simpler algorithm.



                Order the $2k+1$ numbers in increasing order, $a_1leqcdots leq a_{2k+1}$.



                Consider first the $k$ numbers with even indices: each number in this set is weakly smaller than the number immediately after it, which is not in this set. Hence, the sum of these numbers must be at most $1/2$.



                Consider next the $k$ numbers with odd indices, except the last one: again each number in this set is weakly smaller than the number immediately after it, which is not in this set. Hence, the sum of these numbers must be at most $1/2$ too.







                share|cite|improve this answer












                share|cite|improve this answer



                share|cite|improve this answer










                answered Dec 19 '18 at 9:35









                Erel Segal-HaleviErel Segal-Halevi

                4,33011861




                4,33011861























                    1












                    $begingroup$

                    Yes, here's an explicit algorithm for constructing two disjoint $k$-subsets.



                    Let $H$ be a family/multiset of positive numbers indexed by $mathbb{N}_{le(2k+1)}$, with $mathbb{N}$ excluding zero.



                    Let's further constrain $H$ to be weakly monotonically increasing in the index, so $H_{2k+1}$ is the co-largest element.



                    Suppose $H_{2k+1} ge frac{1}{2}$ , then $H_{(1cdots k)}$ and $H_{(k+1 cdots 2k)}$ both have sum $le frac{1}{2}$ and we're done.



                    Suppose $H_{2k+1} le frac{1}{2}$ , then let's define the champion $k$-subfamily $c$ and its set of indices $gamma$ by the following algorithm.




                    1. The very first thing we do is exclude the largest index $2k+1$ (which corresponds to the greatest element of $H$), none of the candidate subsets involve this index.

                    2. Enumerate all the $k$-subsets of $mathbb{N}_{le(2k)}$ , call the subset under consideration $w$

                    3. reject all $w$'s for which $sum_{j in w}H_j$ is greater than or equal to $frac{1}{2}$ .

                    4. reject all $w$'s that do not have the maximum possible sum of any of the remaining $w$'s.

                    5. Of the remaining sets, pick the greatest remaining set by comparing the greatest indices one at a time of $w$ . For instance ${5,4,1}$ beats ${5,3,1}$ because $4 > 3$ . This tiebreaker is completely arbitrary, it's just there to guarantee uniqueness.


                    6. $c$ is $w$ and $gamma$ is $H_c$ .


                    Note that at least one subset $w = {1, cdots, k}$ makes it past step 3, so the algorithm always produces a unique champion subset of indices $c$, which corresponds to a multiset $gamma = H_c$.



                    Let's define multiset indexed by the champion complement set $H[(mathbb{N}le 2*k) setminus c)]$. Henceforth $d = (mathbb{N}le 2*k) setminus c$, and $delta = H_d$ .



                    $gamma$ has sum less than $frac{1}{2}$ by construction. We will show that $delta$ also has sum less than $frac{1}{2}$ .





                    If $sum delta = sum_{j in d}H_j lt frac{1}{2}$, then we have found a pair of disjoint subsets $c$ and $d$ each with sum less than $frac{1}{2}$, as required.





                    If $sum delta = frac{1}{2}$, then then the sum of $sum gamma$ and $H_{2k+1}$ is $frac{1}{2}$ .



                    We will define an operation in terms of $gamma$ and $delta$ to produce a new multiset $gamma'$ with a larger sum than $gamma$, but still less than $frac{1}{2}$ .



                    Pick the largest element of $delta$, tie-breaking on the index if necessary. Call its index $t in d$ . Pick the largest element of $gamma$ less than $t$ , call its index $s in c$ .



                    If there is no $s in c$, then $sum delta le sum gamma$. $sum gamma < frac{1}{2}$ by construction, so there's a contradiction. Therefore there is an $s in c$ less than $t in d$ .



                    $t-s$ is positive by construction. $t-s$ is also less than $H_{2k+1}$ because it is a difference of two positive numbers, each of which is smaller than $H_{2k+1}$ .



                    Define $gamma'$ as the multiset obtained by removing the element with index $s$ from $gamma$ and adding the element with index $t$ from $delta$ . $gamma'$ is a better candidate for champion than $gamma$ . Therefore $gamma$ is not champion.





                    If $sum delta gt frac{1}{2}$, then we can repeat the construction defined above for producing $gamma'$ . $gamma'$ has sum less than $frac{1}{2}$ and is a better candidate for champion set than $gamma$ . Therefore $gamma$ is not champion.






                    share|cite|improve this answer











                    $endgroup$













                    • $begingroup$
                      What is "co-largest"?
                      $endgroup$
                      – Erel Segal-Halevi
                      Dec 19 '18 at 3:59










                    • $begingroup$
                      "one of the largest". $gamma$ and $delta$ are multisets so there might be multiple largest items. It's overloading the co- prefix, but I have heard it before. Is there a better way to say it?
                      $endgroup$
                      – Gregory Nisbet
                      Dec 19 '18 at 4:13


















                    1












                    $begingroup$

                    Yes, here's an explicit algorithm for constructing two disjoint $k$-subsets.



                    Let $H$ be a family/multiset of positive numbers indexed by $mathbb{N}_{le(2k+1)}$, with $mathbb{N}$ excluding zero.



                    Let's further constrain $H$ to be weakly monotonically increasing in the index, so $H_{2k+1}$ is the co-largest element.



                    Suppose $H_{2k+1} ge frac{1}{2}$ , then $H_{(1cdots k)}$ and $H_{(k+1 cdots 2k)}$ both have sum $le frac{1}{2}$ and we're done.



                    Suppose $H_{2k+1} le frac{1}{2}$ , then let's define the champion $k$-subfamily $c$ and its set of indices $gamma$ by the following algorithm.




                    1. The very first thing we do is exclude the largest index $2k+1$ (which corresponds to the greatest element of $H$), none of the candidate subsets involve this index.

                    2. Enumerate all the $k$-subsets of $mathbb{N}_{le(2k)}$ , call the subset under consideration $w$

                    3. reject all $w$'s for which $sum_{j in w}H_j$ is greater than or equal to $frac{1}{2}$ .

                    4. reject all $w$'s that do not have the maximum possible sum of any of the remaining $w$'s.

                    5. Of the remaining sets, pick the greatest remaining set by comparing the greatest indices one at a time of $w$ . For instance ${5,4,1}$ beats ${5,3,1}$ because $4 > 3$ . This tiebreaker is completely arbitrary, it's just there to guarantee uniqueness.


                    6. $c$ is $w$ and $gamma$ is $H_c$ .


                    Note that at least one subset $w = {1, cdots, k}$ makes it past step 3, so the algorithm always produces a unique champion subset of indices $c$, which corresponds to a multiset $gamma = H_c$.



                    Let's define multiset indexed by the champion complement set $H[(mathbb{N}le 2*k) setminus c)]$. Henceforth $d = (mathbb{N}le 2*k) setminus c$, and $delta = H_d$ .



                    $gamma$ has sum less than $frac{1}{2}$ by construction. We will show that $delta$ also has sum less than $frac{1}{2}$ .





                    If $sum delta = sum_{j in d}H_j lt frac{1}{2}$, then we have found a pair of disjoint subsets $c$ and $d$ each with sum less than $frac{1}{2}$, as required.





                    If $sum delta = frac{1}{2}$, then then the sum of $sum gamma$ and $H_{2k+1}$ is $frac{1}{2}$ .



                    We will define an operation in terms of $gamma$ and $delta$ to produce a new multiset $gamma'$ with a larger sum than $gamma$, but still less than $frac{1}{2}$ .



                    Pick the largest element of $delta$, tie-breaking on the index if necessary. Call its index $t in d$ . Pick the largest element of $gamma$ less than $t$ , call its index $s in c$ .



                    If there is no $s in c$, then $sum delta le sum gamma$. $sum gamma < frac{1}{2}$ by construction, so there's a contradiction. Therefore there is an $s in c$ less than $t in d$ .



                    $t-s$ is positive by construction. $t-s$ is also less than $H_{2k+1}$ because it is a difference of two positive numbers, each of which is smaller than $H_{2k+1}$ .



                    Define $gamma'$ as the multiset obtained by removing the element with index $s$ from $gamma$ and adding the element with index $t$ from $delta$ . $gamma'$ is a better candidate for champion than $gamma$ . Therefore $gamma$ is not champion.





                    If $sum delta gt frac{1}{2}$, then we can repeat the construction defined above for producing $gamma'$ . $gamma'$ has sum less than $frac{1}{2}$ and is a better candidate for champion set than $gamma$ . Therefore $gamma$ is not champion.






                    share|cite|improve this answer











                    $endgroup$













                    • $begingroup$
                      What is "co-largest"?
                      $endgroup$
                      – Erel Segal-Halevi
                      Dec 19 '18 at 3:59










                    • $begingroup$
                      "one of the largest". $gamma$ and $delta$ are multisets so there might be multiple largest items. It's overloading the co- prefix, but I have heard it before. Is there a better way to say it?
                      $endgroup$
                      – Gregory Nisbet
                      Dec 19 '18 at 4:13
















                    1












                    1








                    1





                    $begingroup$

                    Yes, here's an explicit algorithm for constructing two disjoint $k$-subsets.



                    Let $H$ be a family/multiset of positive numbers indexed by $mathbb{N}_{le(2k+1)}$, with $mathbb{N}$ excluding zero.



                    Let's further constrain $H$ to be weakly monotonically increasing in the index, so $H_{2k+1}$ is the co-largest element.



                    Suppose $H_{2k+1} ge frac{1}{2}$ , then $H_{(1cdots k)}$ and $H_{(k+1 cdots 2k)}$ both have sum $le frac{1}{2}$ and we're done.



                    Suppose $H_{2k+1} le frac{1}{2}$ , then let's define the champion $k$-subfamily $c$ and its set of indices $gamma$ by the following algorithm.




                    1. The very first thing we do is exclude the largest index $2k+1$ (which corresponds to the greatest element of $H$), none of the candidate subsets involve this index.

                    2. Enumerate all the $k$-subsets of $mathbb{N}_{le(2k)}$ , call the subset under consideration $w$

                    3. reject all $w$'s for which $sum_{j in w}H_j$ is greater than or equal to $frac{1}{2}$ .

                    4. reject all $w$'s that do not have the maximum possible sum of any of the remaining $w$'s.

                    5. Of the remaining sets, pick the greatest remaining set by comparing the greatest indices one at a time of $w$ . For instance ${5,4,1}$ beats ${5,3,1}$ because $4 > 3$ . This tiebreaker is completely arbitrary, it's just there to guarantee uniqueness.


                    6. $c$ is $w$ and $gamma$ is $H_c$ .


                    Note that at least one subset $w = {1, cdots, k}$ makes it past step 3, so the algorithm always produces a unique champion subset of indices $c$, which corresponds to a multiset $gamma = H_c$.



                    Let's define multiset indexed by the champion complement set $H[(mathbb{N}le 2*k) setminus c)]$. Henceforth $d = (mathbb{N}le 2*k) setminus c$, and $delta = H_d$ .



                    $gamma$ has sum less than $frac{1}{2}$ by construction. We will show that $delta$ also has sum less than $frac{1}{2}$ .





                    If $sum delta = sum_{j in d}H_j lt frac{1}{2}$, then we have found a pair of disjoint subsets $c$ and $d$ each with sum less than $frac{1}{2}$, as required.





                    If $sum delta = frac{1}{2}$, then then the sum of $sum gamma$ and $H_{2k+1}$ is $frac{1}{2}$ .



                    We will define an operation in terms of $gamma$ and $delta$ to produce a new multiset $gamma'$ with a larger sum than $gamma$, but still less than $frac{1}{2}$ .



                    Pick the largest element of $delta$, tie-breaking on the index if necessary. Call its index $t in d$ . Pick the largest element of $gamma$ less than $t$ , call its index $s in c$ .



                    If there is no $s in c$, then $sum delta le sum gamma$. $sum gamma < frac{1}{2}$ by construction, so there's a contradiction. Therefore there is an $s in c$ less than $t in d$ .



                    $t-s$ is positive by construction. $t-s$ is also less than $H_{2k+1}$ because it is a difference of two positive numbers, each of which is smaller than $H_{2k+1}$ .



                    Define $gamma'$ as the multiset obtained by removing the element with index $s$ from $gamma$ and adding the element with index $t$ from $delta$ . $gamma'$ is a better candidate for champion than $gamma$ . Therefore $gamma$ is not champion.





                    If $sum delta gt frac{1}{2}$, then we can repeat the construction defined above for producing $gamma'$ . $gamma'$ has sum less than $frac{1}{2}$ and is a better candidate for champion set than $gamma$ . Therefore $gamma$ is not champion.






                    share|cite|improve this answer











                    $endgroup$



                    Yes, here's an explicit algorithm for constructing two disjoint $k$-subsets.



                    Let $H$ be a family/multiset of positive numbers indexed by $mathbb{N}_{le(2k+1)}$, with $mathbb{N}$ excluding zero.



                    Let's further constrain $H$ to be weakly monotonically increasing in the index, so $H_{2k+1}$ is the co-largest element.



                    Suppose $H_{2k+1} ge frac{1}{2}$ , then $H_{(1cdots k)}$ and $H_{(k+1 cdots 2k)}$ both have sum $le frac{1}{2}$ and we're done.



                    Suppose $H_{2k+1} le frac{1}{2}$ , then let's define the champion $k$-subfamily $c$ and its set of indices $gamma$ by the following algorithm.




                    1. The very first thing we do is exclude the largest index $2k+1$ (which corresponds to the greatest element of $H$), none of the candidate subsets involve this index.

                    2. Enumerate all the $k$-subsets of $mathbb{N}_{le(2k)}$ , call the subset under consideration $w$

                    3. reject all $w$'s for which $sum_{j in w}H_j$ is greater than or equal to $frac{1}{2}$ .

                    4. reject all $w$'s that do not have the maximum possible sum of any of the remaining $w$'s.

                    5. Of the remaining sets, pick the greatest remaining set by comparing the greatest indices one at a time of $w$ . For instance ${5,4,1}$ beats ${5,3,1}$ because $4 > 3$ . This tiebreaker is completely arbitrary, it's just there to guarantee uniqueness.


                    6. $c$ is $w$ and $gamma$ is $H_c$ .


                    Note that at least one subset $w = {1, cdots, k}$ makes it past step 3, so the algorithm always produces a unique champion subset of indices $c$, which corresponds to a multiset $gamma = H_c$.



                    Let's define multiset indexed by the champion complement set $H[(mathbb{N}le 2*k) setminus c)]$. Henceforth $d = (mathbb{N}le 2*k) setminus c$, and $delta = H_d$ .



                    $gamma$ has sum less than $frac{1}{2}$ by construction. We will show that $delta$ also has sum less than $frac{1}{2}$ .





                    If $sum delta = sum_{j in d}H_j lt frac{1}{2}$, then we have found a pair of disjoint subsets $c$ and $d$ each with sum less than $frac{1}{2}$, as required.





                    If $sum delta = frac{1}{2}$, then then the sum of $sum gamma$ and $H_{2k+1}$ is $frac{1}{2}$ .



                    We will define an operation in terms of $gamma$ and $delta$ to produce a new multiset $gamma'$ with a larger sum than $gamma$, but still less than $frac{1}{2}$ .



                    Pick the largest element of $delta$, tie-breaking on the index if necessary. Call its index $t in d$ . Pick the largest element of $gamma$ less than $t$ , call its index $s in c$ .



                    If there is no $s in c$, then $sum delta le sum gamma$. $sum gamma < frac{1}{2}$ by construction, so there's a contradiction. Therefore there is an $s in c$ less than $t in d$ .



                    $t-s$ is positive by construction. $t-s$ is also less than $H_{2k+1}$ because it is a difference of two positive numbers, each of which is smaller than $H_{2k+1}$ .



                    Define $gamma'$ as the multiset obtained by removing the element with index $s$ from $gamma$ and adding the element with index $t$ from $delta$ . $gamma'$ is a better candidate for champion than $gamma$ . Therefore $gamma$ is not champion.





                    If $sum delta gt frac{1}{2}$, then we can repeat the construction defined above for producing $gamma'$ . $gamma'$ has sum less than $frac{1}{2}$ and is a better candidate for champion set than $gamma$ . Therefore $gamma$ is not champion.







                    share|cite|improve this answer














                    share|cite|improve this answer



                    share|cite|improve this answer








                    edited Dec 19 '18 at 4:26

























                    answered Dec 18 '18 at 22:54









                    Gregory NisbetGregory Nisbet

                    714612




                    714612












                    • $begingroup$
                      What is "co-largest"?
                      $endgroup$
                      – Erel Segal-Halevi
                      Dec 19 '18 at 3:59










                    • $begingroup$
                      "one of the largest". $gamma$ and $delta$ are multisets so there might be multiple largest items. It's overloading the co- prefix, but I have heard it before. Is there a better way to say it?
                      $endgroup$
                      – Gregory Nisbet
                      Dec 19 '18 at 4:13




















                    • $begingroup$
                      What is "co-largest"?
                      $endgroup$
                      – Erel Segal-Halevi
                      Dec 19 '18 at 3:59










                    • $begingroup$
                      "one of the largest". $gamma$ and $delta$ are multisets so there might be multiple largest items. It's overloading the co- prefix, but I have heard it before. Is there a better way to say it?
                      $endgroup$
                      – Gregory Nisbet
                      Dec 19 '18 at 4:13


















                    $begingroup$
                    What is "co-largest"?
                    $endgroup$
                    – Erel Segal-Halevi
                    Dec 19 '18 at 3:59




                    $begingroup$
                    What is "co-largest"?
                    $endgroup$
                    – Erel Segal-Halevi
                    Dec 19 '18 at 3:59












                    $begingroup$
                    "one of the largest". $gamma$ and $delta$ are multisets so there might be multiple largest items. It's overloading the co- prefix, but I have heard it before. Is there a better way to say it?
                    $endgroup$
                    – Gregory Nisbet
                    Dec 19 '18 at 4:13






                    $begingroup$
                    "one of the largest". $gamma$ and $delta$ are multisets so there might be multiple largest items. It's overloading the co- prefix, but I have heard it before. Is there a better way to say it?
                    $endgroup$
                    – Gregory Nisbet
                    Dec 19 '18 at 4:13




















                    draft saved

                    draft discarded




















































                    Thanks for contributing an answer to Mathematics Stack Exchange!


                    • Please be sure to answer the question. Provide details and share your research!

                    But avoid



                    • Asking for help, clarification, or responding to other answers.

                    • Making statements based on opinion; back them up with references or personal experience.


                    Use MathJax to format equations. MathJax reference.


                    To learn more, see our tips on writing great answers.




                    draft saved


                    draft discarded














                    StackExchange.ready(
                    function () {
                    StackExchange.openid.initPostLogin('.new-post-login', 'https%3a%2f%2fmath.stackexchange.com%2fquestions%2f3045731%2ffinding-two-disjoint-subsets-whose-sum-is-below-1-2%23new-answer', 'question_page');
                    }
                    );

                    Post as a guest















                    Required, but never shown





















































                    Required, but never shown














                    Required, but never shown












                    Required, but never shown







                    Required, but never shown

































                    Required, but never shown














                    Required, but never shown












                    Required, but never shown







                    Required, but never shown







                    Popular posts from this blog

                    How do I know what Microsoft account the skydrive app is syncing to?

                    When does type information flow backwards in C++?

                    Grease: Live!